Hard Verses: Not Your Namby Pamby Bible Study. These Bible Verses are Hard!

Hard Verses: Not Your Namby Pamby Bible Study. These Bible Verses are Hard!

Follow Hard Verses: Not Your Namby Pamby Bible Study. These Bible Verses are Hard!
Share on
Copy link to clipboard

In a world of namby pamby Bible studies, there stands a man, bold and courageous enough to take on the most difficult of Bible passages.

Bryan Cook


    • Apr 9, 2018 LATEST EPISODE
    • infrequent NEW EPISODES
    • 22m AVG DURATION
    • 35 EPISODES


    Search for episodes from Hard Verses: Not Your Namby Pamby Bible Study. These Bible Verses are Hard! with a specific topic:

    Latest episodes from Hard Verses: Not Your Namby Pamby Bible Study. These Bible Verses are Hard!

    Episode 33: Laying on of the Apostle's hands (Acts 8:18)

    Play Episode Listen Later Apr 9, 2018 21:13


    We explore the expression concerning Simon the sorcerer in Acts 8.  We also go into the imparting of miracles and the time frame for these miracles.

    Episode 32: Finalizing Matthew 24

    Play Episode Listen Later Apr 4, 2018 27:45


    This is the final installment for Matthew 24 as it relates to the End Time prophecies.  I am needing more hard verses for the show.  Please send in your suggestions.  Also, make this podcast successful by sharing and subscribing. Thanks for all the support and encouragement. Bryan

    Episode 31 Matthew 24 continued...

    Play Episode Listen Later Mar 26, 2018 32:12


    I know this is taking several episodes but consider that this is an entire chapter.  We will go back to the old format after this series on End Times and the destruction of Jerusalem in AD70.  

    Episode 30: Matthew 24 with some more background

    Play Episode Listen Later Mar 12, 2018 14:25


    I am using Burton Coffman's commentary on Matthew 24 to give the background to the scene in chapter 24. This is available for free to use at studylight.org.  Just do a Google search for Burton Coffman commentary on Matthew and it comes right up.  Let me know about the slower format.

    Episode 29: Matthew 24 End Times? Part 2

    Play Episode Listen Later Mar 5, 2018 28:43


    This section along with Luke 21 are speaking about the end of certain things.  What are these things?  The last days, the second coming, the destruction of the temple or Jerusalem?  We look at these verses and also at some of the experts in the field (note that I do not agree with the experts most of the time).

    Episode 28: Matthew 24 and Luke 21...End Times?

    Play Episode Listen Later Feb 26, 2018 29:44


    Hey Hard Verses listeners. Hope all is well.  Today's show is a break of form so to speak. Instead of a hard verse, we are taking on some hard chapters (Matthew 24 and Luke 21).  These two chapters are concerned with several things. Jesus will tell of the destruction of the temple all the way to the end of the world.  How can we tell which part is talking about what?  That is our discussion on this and subsequent shows.  Be sure and support the show by subscribing, commenting, sharing and even asking questions on the Youtube and other platforms.  Thanks to all the new subscribers.  This is going to be a great year for the show if you guys continue to let the word out.  Talk to you soon. Bryan

    Ep. 27. Went Down from Jerusalem.

    Play Episode Listen Later Feb 5, 2018 8:49


    This is a hard verse short.  This concept of going down to a place had very little to do with North and South and more to do with elevation.  We look at several verses that have this issue. Go check out the Bible Genius Bible Study now available for download and free to use for any size class.  Acts and Proverbs are the two books in the hopper. Thanks for listening. Bryan

    Dash your Little Ones on the Stones. Psalms 127:9. Ep. 26

    Play Episode Listen Later Jan 29, 2018 24:35


    This is a closer look at Psalms 127:9 concerning the phrase, " happy are they that dash their little ones on the stones."  I enlist the commentators of Barnes, Coffman, Clarke, and K&D.  For those that are not going to listen, just know that Edom and Babylon are going to get theirs. Thanks to all the subscribers and supporters.  I pray 2018 will be a great year for us all. Bryan

    Eli Eli Lama Sabachthani while on the cross in Matthew 27:46.

    Play Episode Listen Later Jan 22, 2018 15:58


    Eli Eli Lama Sabachthani, My God, My God, why hast thou forsaken me? Matthew 27:46. Did Jesus not really know what was going on? How does Psalm 22 relate?  We shall find out on this episode of the Hard Verses Podcast.

    EP. 24 The Man of Perdition: 2 Thess. 2:3

    Play Episode Listen Later Jan 15, 2018 35:30


    This is a great hard verse for the Bible Genius to go after.  I thank Pete Davis for the research and love of the truth and his contribution to the podcast and to my life.   One of the resources that was heavily relied on is this article by Lee. http://www.historicism.net/readingmaterials/IIThess.pdf I can neither endorse or denounce but did like his historical perspective on this verse. For more information go to the website preacherbryan.com. Keep up the study.  Thanks for stopping by. In Christian love,  Bryan

    Episode 23: Man of Perdition --but not really.

    Play Episode Listen Later Jan 12, 2018 15:35


    This is my confession that this verse is harder than originally thought.  I am still in the works and should have some information by next week. My plan is to have the answers to the proposed questions concerning the "man of perdition" from 2 Thess. 2.  Thanks for all the encouragement and comments.  The video of the livestream from this content is up at Youtube.  Look for "Hard Verses" you should see me and my parrot mug. Love, Bryan

    Episode 22: The Antichrist. 1 John giving us the full description.

    Play Episode Listen Later Jan 6, 2018 17:56


    Spoiler alert:  It is not the man of perdition in 2 Thess. You are probably wanting to hear about that instead.  Thanks to everyone who has been so patient during my wife's illness.  Yes it has been a while.   In Christian love, Bryan Cook

    Minisode (22b): The body is the temple of God, 1 Corinthians 6:19

    Play Episode Listen Later Jan 6, 2018 19:52


    Special thanks to the students at NWFSBS.  They did a great job being in the studio.

    Hard Verses. Episode 21, A Sin Unto Death--1 John 5:16,17

    Play Episode Listen Later Jul 29, 2017 24:37


    Thanks to Jeff Orr and Guyton Montgomery for helping out this episode. The hard verse is 1 John 5:16,17, talking of a sin unto death.  What is a sin that is unto death? What is a sin not unto death?  Are there sins that we are not to pray?  We discuss all this on todays episode.   Please remember to subscribe and comment and like and support and share and.... Video is on the Youtube channel.

    Sons of God in Job 1:6. Hard Verses Episode 20

    Play Episode Listen Later Mar 16, 2017 22:01


    Thanks to Guyton Montgomery for helping with the show. These are the notes used in the show: 20170315 Job 1:6 Job 1:5  And it was so, when the days of their feasting were gone about, that Job sent and sanctified them, and rose up early in the morning, and offered burnt offerings according to the number of them all: for Job said, It may be that my sons have sinned, and cursed God in their hearts. Thus did Job continually. Satan Allowed to Test Job Job 1:6  Now there was a day when the sons of God came to present themselves before the LORD, and Satan came also among them. Job 1:7  And the LORD said unto Satan, Whence comest thou? Then Satan answered the LORD, and said, From going to and fro in the earth, and from walking up and down in it. Job 1:8  And the LORD said unto Satan, Hast thou considered my servant Job, that there is none like him in the earth, a perfect and an upright man, one that feareth God, and escheweth evil? Burton Coffman Job 1:6-12 THAT CONVERSATION BETWEEN GOD AND SATAN Now it came to pass on the day when the sons of God came to present themselves before Jehovah, that Satan also came among them. And Jehovah said unto Satan, Whence comest thou? Then Satan answered Jehovah, and said, From going to and fro in the earth, and from walking up and down in it. And Jehovah said unto Satan, Hast thou considered my servant Job? for there is none like him in the earth, a perfect and an upright man, one that feareth God, and turneth away from evil. Then Satan answered Jehovah, and said, Doth Job fear God for nought? Hast not thou made a hedge about him, and about his house, and about all that he hath, on every side? thou hast blessed the work of his hands, and his substance is increased in the land. But put forth thy hand now, and touch all that he hath, and he will renounce thee to thy face. And Jehovah said unto Satan, Behold, all that he hath is in thy power; only upon himself put not forth thy hand. So Satan went forth from the presence of Jehovah. See our chapter introduction for the location of this scene, not in heaven, but upon earth when God’s people were assembled for worship. Jehovah. Jehovah ... Jehovah ... Jehovah ... Jehovah ... Jehovah ... Jehovah (Job 1:6-12). Seven times in six verses, the covenant name of God appears in this passage, thundering the message that the sons of God here are those in covenant relationship with the Eternal. Note that [~’Elohiym] (God) is used twice in the preceding paragraph. Are the holy angels within any such covenant relationship? Absolutely, NO! How blind the critics are to this passage! Dummelow noted that the use of the Tetragrammaton here is thought by some, To be a slip on the part of the author! F6 How could anybody make a slip seven times in six verses? Our own opinion is that we have proof here, as Dummelow wrote, that, The Tetragrammaton (Jehovah) may have been much older than Moses. F7 This writer fully believes that the name Jehovah is indeed much older than Moses. (See my excursus on the Tetragrammaton (Y-H-W-H), beginning on p. 32 of my commentary on Exodus.) "Satan here has ready access to heaven." F8 See chapter introduction for many reasons why such a comment is erroneous. "The sons of God ... are divine beings who shared in the nature of divinity." F9 Rowley enunciated the same error, "Sons of God means superhuman, or celestial beings." F10 "To interpret these words thus is to suppose that the very same words in Gen 6:1-3 teach that angels intermarried with men producing a race of giants, despite the fact that the Christ himself declared that angels "Neither marry nor are given in marriage" (Mat 22:30). The true meaning of Sons of God is found in Rom 8:14. How are they sons of God? "They are born of God, born of water and the Spirit". In short, they are followers and worshippers of God; and so were those mentioned in Gen 6:1-3, and in this passage before us. To suppose that the Holy Spirit in this passage was speaking of angels is ridiculous. Did He not know the word angels? If that is what he meant, why did not the Holy Spirit write angels? This student of God’s Word has had his fill, and more, of the speculations of commentators who presume to tell us what the Holy Spirit meant, instead of what he said. Rawlinson, usually quite accurate, misses it here, stating that, "Job 1:7 teaches that Satan, therefore, is not, like most of his fallen angels, "reserved in chains under darkness to the judgment of the last day (Jud 1:6)." F11 However, this very passage teaches that Satan is chained. (See more on the chaining of Satan under Job 1:12). He could not even touch Job, without God’s permission; and, even after the Lord gave him permission to destroy Job’s possessions, he was not allowed to touch the person of Job (Job 1:12). Barnes Job 1:6 There was a day when the sons of God - All the versions, and indeed all the critics, are puzzled with the phrase sons of God; בני האלהים beney haelohim, literally, sons of the God, or sons of the gods. The Vulgate has simply filii dei, sons of God. The Septuagint, οἱ αγγελοι του θεου, the angels of God. The Chaldee, כתי מלאכיא kittey malachaiya, troops of angels. The Syriac retains the Hebrew words and letters, only leaving out the demonstrative ה he in the word האלהים haelohim, thus, (Syriac) baney Elohim. JFB sons of God — angels (Job 38:7; 1Ki 22:19). They present themselves to render account of their “ministry” in other parts of the universe (Heb 1:14).

    Going on Hiatus from Podcast.

    Play Episode Listen Later Oct 1, 2016 4:54


    I will be going on hiatus from the show for a little bit. I will be getting caught up on my Master's degree and paying off some bills. Please keep in touch on social media. I will be back. Still send in suggestions.

    You are a Bloody Husband; Exodus 4:24-26

    Play Episode Listen Later Sep 6, 2016 25:07


    Hard Verses Episode 18: You are a Bloody Husband; Exodus 4:24-26. The Problem. Exo 4:20 And Moses took his wife and his sons, and set them upon an ass, and he returned to the land of Egypt: and Moses took the rod of God in his hand. Exo 4:21 And the LORD said unto Moses, When thou goest to return into Egypt, see that thou do all those wonders before Pharaoh, which I have put in thine hand: but I will harden his heart, that he shall not let the people go. Exo 4:22 And thou shalt say unto Pharaoh, Thus saith the LORD, Israel is my son, even my firstborn: Exo 4:23 And I say unto thee, Let my son go, that he may serve me: and if thou refuse to let him go, behold, I will slay thy son, even thy firstborn. Exo 4:24 And it came to pass by the way in the inn, that the LORD met him, and sought to kill him. Exo 4:25 Then Zipporah took a sharp stone, and cut off the foreskin of her son, and cast it at his feet, and said, Surely a bloody husband art thou to me. Exo 4:26 So he let him go: then she said, A bloody husband thou art, because of the circumcision. 1. What is going on here? 2. Moses seems to be sending his family away while he is going to prepare to go to Egypt. 3. The Lord sought to kill him. Who is him and why does God want to kill him? 4. What is the deal with Zipporah circumcising her son and calling her husband a bloody husband? The Solution. Exo 4:20 And Moses took his wife and his sons, and set them upon an ass, and he returned to the land of Egypt: and Moses took the rod of God in his hand. Exo 4:21 And the LORD said unto Moses, When thou goest to return into Egypt, see that thou do all those wonders before Pharaoh, which I have put in thine hand: but I will harden his heart, that he shall not let the people go. Exo 4:22 And thou shalt say unto Pharaoh, Thus saith the LORD, Israel is my son, even my firstborn: Exo 4:23 And I say unto thee, Let my son go, that he may serve me: and if thou refuse to let him go, behold, I will slay thy son, even thy firstborn. Exo 4:24 And it came to pass by the way in the inn, that the LORD met him, and sought to kill him. Exo 4:25 Then Zipporah took a sharp stone, and cut off the foreskin of her son, and cast it at his feet, and said, Surely a bloody husband art thou to me. Exo 4:26 So he let him go: then she said, A bloody husband thou art, because of the circumcision. 1. What is going on? Moses just met God at the burning bush in chapter four. He has met with Jethro his father-in-law and asked to be released to go let God’s people go vs. 18. Now he is on his way to Egypt. 2. Where did his family go? Moses set me on an ass (Septuagint states beasts, Hebrew idiom, the ass to mean even the plural of beasts.) or that Zipporah and baby were riding while Gershom led the beast. It would seem that they stayed at the inn before heading back to Jethro. Ex. 18:2-5 speaks of Jethro bringing the family out to Moses in the wilderness. 3. The Lord sought to kill him. Who is him and why does God want to kill him? It seems that Moses was the one stricken by some severe sickness. Zipporah in seeing this, realized that it was because one of her children was uncircumcised and picked up a sharp rock and circumcised the child and threw it at the feet of Moses on which the Lord “let him go.” 4. What is the deal with Zipporah circumcising her son and calling her husband a bloody husband? It would seem that Moses was unable to perform the procedure and worrying that her husband was about to die, picked up the rock and did a makeshift circumcision. It was probably from her disdain of the procedure that it was not done. This would be the reason for the outburst of the bloody husband remark. Gen 17 actually records the first circumcision with Abram and his household. It states that Abram circumcised everyone, young and old and even himself. It does not necessarily say that Abraham did that to himself. The lesson for us today 1. Don’t wait to do what God wants you to do. 2. Even if things are going to be uncomfortable, if God requires it, do it. 3. God is serious about obeying His command. He was going to kill Moses. 2Th 1:7 And to you who are troubled rest with us, when the Lord Jesus shall be revealed from heaven with his mighty angels, 2Th 1:8 In flaming fire taking vengeance on them that know not God, and that obey not the gospel of our Lord Jesus Christ: 2Th 1:9 Who shall be punished with everlasting destruction from the presence of the Lord, and from the glory of his power; Tools I Used Burton Coffman (e-sword module) Albert Barnes (e-sword module) Adam Clarke (e-sword module) Jamieson Faucet and Brown (e-sword module) Keil and Delitzsch (e-sword module) Call to action. Subscribe and rate. You can also write a review. (Itunes). There are also the podcasts available on Youtube, look for “hard verses” with some additional videos besides the podcast.

    Abstain from Meats Offered to Idols, Acts 15:29

    Play Episode Listen Later Sep 1, 2016 22:08


    Hard Verses Episode 17: Abstain from Meats, Acts 15:29 Welcome to the Hard Verses podcast. Episode 17: Abstain from Meats, Acts 15:29 Hard verses is a podcast not afraid to take on the harder verses in the Bible.  Each week we find difficult verses in God's word. The Problem. Act 15:29  That ye abstain from meats offered to idols, and from blood, and from things strangled, and from fornication: from which if ye keep yourselves, ye shall do well. Fare ye well. Are we to abstain from eating meats offered to idols? Other verses would indicate that a strong Christian can eat meats offered to idols. Why the change? 1Co 10:19  What say I then? that the idol is any thing, or that which is offered in sacrifice to idols is any thing? 1Co 10:25  Whatsoever is sold in the shambles, that eat, asking no question for conscience sake: 1Co 10:26  For the earth is the Lord's, and the fulness thereof. 1Co 10:27  If any of them that believe not bid you to a feast, and ye be disposed to go; whatsoever is set before you, eat, asking no question for conscience sake. 1Co 10:28  But if any man say unto you, This is offered in sacrifice unto idols, eat not for his sake that shewed it, and for conscience sake: for the earth is the Lord's, and the fulness thereof: 1Co 10:29  Conscience, I say, not thine own, but of the other: for why is my liberty judged of another man's conscience? 1Co 10:30  For if I by grace be a partaker, why am I evil spoken of for that for which I give thanks? 1Co 10:31  Whether therefore ye eat, or drink, or whatsoever ye do, do all to the glory of God.   Are Christians still not supposed to eat blood or things strangled?   The Solution. Act 15:29  That ye abstain from meats offered to idols, and from blood, and from things strangled, and from fornication: from which if ye keep yourselves, ye shall do well. Fare ye well. This would be indicative of what was instructed to the Gentiles minus all the junk that the Jews were trying to put in there. The idol meat. The meats offered to idols would have been the remaining meat from the sacrifice of the pagan priests.  The meat would go either back to the worshipper and to the priest to sell at the market (Barclay 1976, 116). Barclay would go on to say that “No Christian must risk pollution by eating such meat for it had been offered to an idol” (ibid). The strangled and the blood. The Jews would have looked at animals strangled and not drained properly of blood as unclean.  God had said life is in the blood (Lev 17:11).  Barclay would go on to indicate that this act would make it possible for Jew and Gentile to be able to come together The law of eating blood predates the law of Moses.   Gen 9:3  Every moving thing that liveth shall be meat for you; even as the green herb have I given you all things. Gen 9:4  But flesh with the life thereof, which is the blood thereof, shall ye not eat. Gen 9:5  And surely your blood of your lives will I require; at the hand of every beast will I require it, and at the hand of man; at the hand of every man's brother will I require the life of man.   And is also shown to be inspired instruction for the church. Act 15:28  For it seemed good to the Holy Ghost, and to us, to lay upon you no greater burden than these necessary things; Act 15:29  That ye abstain from meats offered to idols, and from blood, and from things strangled, and from fornication: from which if ye keep yourselves, ye shall do well. Fare ye well.   It is understood that this would make things better between Jew and Gentile, since blood and idol feast offerings would be something of an abomination to a Jew. What about those verses that say that is nothing to it? 1Co 10:19  What say I then? that the idol is any thing, or that which is offered in sacrifice to idols is any thing? 1Co 10:25  Whatsoever is sold in the shambles, that eat, asking no question for conscience sake: 1Co 10:26  For the earth is the Lord's, and the fulness thereof. 1Co 10:27  If any of them that believe not bid you to a feast, and ye be disposed to go; whatsoever is set before you, eat, asking no question for conscience sake. 1Co 10:28  But if any man say unto you, This is offered in sacrifice unto idols, eat not for his sake that shewed it, and for conscience sake: for the earth is the Lord's, and the fulness thereof: 1Co 10:29  Conscience, I say, not thine own, but of the other: for why is my liberty judged of another man's conscience? 1Co 10:30  For if I by grace be a partaker, why am I evil spoken of for that for which I give thanks? 1Co 10:31  Whether therefore ye eat, or drink, or whatsoever ye do, do all to the glory of God.   It seems that the market place, one would not know if it had been offered to idols or not. Also if you were invited to a house, there would be no way to know if it had been idol food.     The lesson for us today. Don’t add to or take away from God’s word. Don’t do things that will cause others to stumble. Take some time on the hard verses and dig until you get a decent answer. Don’t skip. Use your resources. Tools I Used Albert Barnes (e-sword module). Barclay, William. The Acts of the Apostles, rev. ed. Philadelphia: Westminster (1976). Burton Coffman Commentary (e-sword module) https://www.apologeticspress.org/apcontent.aspx?category=11&article=2276   Call to action. If you are lost concerning Mosaic law and law with Noah, and the New Testament law, you need to get and do my Bible Genius Study on Acts- all the chapters, you will understand the laws.  Free on preacherbryan.com. Subscribe and rate.   You can also write a review. (Itunes).   There are also the podcasts available on Youtube, look for “hard verses” with some additional videos besides the podcast.  

    It Is Serious Now. Bible Genius--Acts Chapter 1 Now Up.

    Play Episode Listen Later Aug 24, 2016 0:01


    Best Bible study guaranteed or your money back!  How can I make such a claim?  Because it is also free! It is definitely worth every penny. Download link below post of PDF. Link is right HERE Thanks, Preacher Bryan

    2 for 1 Deal: The Mustard Seed and The Leaven Parables

    Play Episode Listen Later Aug 23, 2016 23:41


    Hard Verses Episode 16: Two for One Deal (Matt 13:31-33) Parable of the mustard seed Parable of the leaven The Problem. Mat 13:31 Another parable put he forth unto them, saying, The kingdom of heaven is like to a grain of mustard seed, which a man took, and sowed in his field: Mat 13:32 Which indeed is the least of all seeds: but when it is grown, it is the greatest among herbs, and becometh a tree, so that the birds of the air come and lodge in the branches thereof. Mat 13:33 Another parable spake he unto them; The kingdom of heaven is like unto leaven, which a woman took, and hid in three measures of meal, till the whole was leavened. So you have the mustard seed leaven The Solution. Mat 13:31 Another parable put he forth unto them, saying, The kingdom of heaven is like to a grain of mustard seed, which a man took, and sowed in his field: Mat 13:32 Which indeed is the least of all seeds: but when it is grown, it is the greatest among herbs, and becometh a tree, so that the birds of the air come and lodge in the branches thereof. Mat 13:33 Another parable spake he unto them; The kingdom of heaven is like unto leaven, which a woman took, and hid in three measures of meal, till the whole was leavened. There is nothing before to indicate what this means except the parable of the wheat and tares and the parable of the sower. These are very negative parables. There will be a great number that will die and go to hell. It is quite possible that Jesus needed to balance these negative messages with a couple of positives. 1. The Mustard Seed. The kingdom of heaven is like a grain of mustard seed. The man took and sowed it and it grew. Looked like a tiny, tiny thing but grew into a great 15 foot bush. The last parable, the seed was the word of God. ¾ did not receive the benefit of the word. This is showing that we sow the word. Seems like a small thing but great things can happen. The birds are able to perch. There should be benefit for all those around that receive the word. We know people that were on the wrong path and heard the gospel and everyone is glad they did. Fathers come home. Mothers become mothers. Friends become real friends that we can lean on. Notice how the kingdom grew in the New Testament with such a small beginning. 120, 3000, to the uttermost parts of the world. 2. Leaven Leaven is yeast. The Jews were required to clean their house a week prior to the day of unleavened bread because just a little is contaminate the bread. Often leaven is a bad thing. Bad influence Mat 16:5 And when his disciples were come to the other side, they had forgotten to take bread. Mat 16:6 Then Jesus said unto them, Take heed and beware of the leaven of the Pharisees and of the Sadducees. Mat 16:7 And they reasoned among themselves, saying, It is because we have taken no bread. Mat 16:8 Which when Jesus perceived, he said unto them, O ye of little faith, why reason ye among yourselves, because ye have brought no bread? Mat 16:9 Do ye not yet understand, neither remember the five loaves of the five thousand, and how many baskets ye took up? Mat 16:10 Neither the seven loaves of the four thousand, and how many baskets ye took up? Mat 16:11 How is it that ye do not understand that I spake it not to you concerning bread, that ye should beware of the leaven of the Pharisees and of the Sadducees? Mat 16:12 Then understood they how that he bade them not beware of the leaven of bread, but of the doctrine of the Pharisees and of the Sadducees. 1Co 5:1 It is reported commonly that there is fornication among you, and such fornication as is not so much as named among the Gentiles, that one should have his father's wife. 1Co 5:2 And ye are puffed up, and have not rather mourned, that he that hath done this deed might be taken away from among you. 1Co 5:3 For I verily, as absent in body, but present in spirit, have judged already, as though I were present, concerning him that hath so done this deed, 1Co 5:4 In the name of our Lord Jesus Christ, when ye are gathered together, and my spirit, with the power of our Lord Jesus Christ, 1Co 5:5 To deliver such an one unto Satan for the destruction of the flesh, that the spirit may be saved in the day of the Lord Jesus. 1Co 5:6 Your glorying is not good. Know ye not that a little leaven leaveneth the whole lump? 1Co 5:7 Purge out therefore the old leaven, that ye may be a new lump, as ye are unleavened. For even Christ our passover is sacrificed for us: 1Co 5:8 Therefore let us keep the feast, not with old leaven, neither with the leaven of malice and wickedness; but with the unleavened bread of sincerity and truth. That is bad leaven. This is good leaven. We must become good leaven. Mix us into a crowd and we will affect the whole loaf. The lesson for us today. From the mustard seed. Do the small things and expect God to make the difference. From the leaven Don’t lose heart, you come back and the loaf will have grown. Do your part to be a leavening influence. Tools I Used Last week’s sermon Call to action. Subscribe and rate. You can also write a review. (Itunes). Check out the new Bible Study. “The Bible Genius Bible Study” now available for free on preacherbryan.com There are also the podcasts available on Youtube, look for “hard verses” with some additional videos besides the podcast.

    Baptism for the Dead, 1 Cor. 15:29

    Play Episode Listen Later Aug 16, 2016 20:36


    Hard Verses Episode 15: Baptism for the Dead; 1 Cor. 15:29 Thanks to Patrick Peters for the suggestion. The Problem. 1Co 15:29 Else what shall they do which are baptized for the dead, if the dead rise not at all? why are they then baptized for the dead? What does it mean when it says they are baptized for the dead? Does that mean that you can be baptized for those that have already died? Prophet Joseph Smith first taught about the ordinance of baptism for the dead during a funeral sermon in August 1840. He read much of 1 Corinthians 15, including verse 29, and announced that the Lord would permit Church members to be baptized in behalf of their friends and relatives who had departed this life. He told them “the plan of salvation was calculated to save all who were willing to obey the requirements of the law of God” (Journal History of the Church, 15 Aug. 1840). https://www.mormon.org/faq/baptism-for-the-dead Proxie baptism maybe. Mormons use this as proof. Can we save those that have gone on before and were not living right? The Solution. Let me give the context, let us read some. 1Co 15:20 But now is Christ risen from the dead, and become the firstfruits of them that slept. 1Co 15:21 For since by man came death, by man came also the resurrection of the dead. 1Co 15:22 For as in Adam all die, even so in Christ shall all be made alive. 1Co 15:23 But every man in his own order: Christ the firstfruits; afterward they that are Christ's at his coming. 1Co 15:24 Then cometh the end, when he shall have delivered up the kingdom to God, even the Father; when he shall have put down all rule and all authority and power. 1Co 15:25 For he must reign, till he hath put all enemies under his feet. 1Co 15:26 The last enemy that shall be destroyed is death. 1Co 15:27 For he hath put all things under his feet. But when he saith all things are put under him, it is manifest that he is excepted, which did put all things under him. 1Co 15:28 And when all things shall be subdued unto him, then shall the Son also himself be subject unto him that put all things under him, that God may be all in all. 1Co 15:29 Else what shall they do which are baptized for the dead, if the dead rise not at all? why are they then baptized for the dead? 1Co 15:30 And why stand we in jeopardy every hour? 1Co 15:31 I protest by your rejoicing which I have in Christ Jesus our Lord, I die daily. 1Co 15:32 If after the manner of men I have fought with beasts at Ephesus, what advantageth it me, if the dead rise not? let us eat and drink; for to morrow we die. 1Co 15:33 Be not deceived: evil communications corrupt good manners. 1Co 15:34 Awake to righteousness, and sin not; for some have not the knowledge of God: I speak this to your shame. 1Co 15:35 But some man will say, How are the dead raised up? and with what body do they come? 1Co 15:36 Thou fool, that which thou sowest is not quickened, except it die: 1Co 15:37 And that which thou sowest, thou sowest not that body that shall be, but bare grain, it may chance of wheat, or of some other grain: 1Co 15:38 But God giveth it a body as it hath pleased him, and to every seed his own body. Let us not lose the meaning of the passage. Paul is teaching that there will be a resurrection of the dead. There was doubt about this resurrection. The pagans believed that there was corruptness in the body. This meant that purity was to come by shedding this body. Paul was using this practice of baptism (whatever that meant) to teach about the resurrection of the body. Many have suggested that this practice is lost to antiquity because we have nothing else to describe it. He was not teaching that we were to be baptized for the dead. This was just illustrative (Guy N. Woods). Notice the “they” instead of inclusive words like “we” or “us” which would indicate an outside group. Can you be baptized for someone else? Can you proxie other areas of the faith? Hearing the word of God? Believing/faith in that word and in God? Confessing Jesus before men? Repenting of sins? This just doesn’t make sense. So to make baptism go to that area doesn’t make sense. What could it mean? Could have been a practice of the surrounding pagans. Howard Winters (Commentary on First Corinthians, Practical and Explanatory pg. 217) puts it in a modern day saying, “Else what shall they do who violently attack others in order to end violence, if non-violence is their method ? Why then are they using violence?” Burton Coffman states the following: "The Indians bury a dog and a spear with the fallen warrior; and why should they do that, if there is no resurrection?" Both statements use illustrations without condoning the practice used in the example. One of the safest ways to interpret the passage, which lines up with every major doctrine in the NT is the following: The “they” are Christians that were baptized for their dead spiritual condition. To ask, “why are you being baptized to a spiritual death if you are not going to have a spiritual resurrection?” The lesson for us today. 1. Don’t create doctrine out of a single verse, especially out of its context. 2. Don’t take just part of a teaching, take it all. Tools I Used E-sword (Burton Coffman module). Commentary on the First Corinthians, Practical and Explanatory, Howard Winters. Seek the Old Paths, A study of the Church at Corinth, ed. Garland M. Robinson. Questions and Answers, Open Forum, Freed-Hardeman College Lectures, by Guy N. Woods. https://www.mormon.org/faq/baptism-for-the-dead Call to action. Subscribe and rate. You can also write a review. (Itunes). Check out the new Bible Study. “The Bible Genius Bible Study” now available for free on preacherbryan.com There are also the podcasts available on Youtube, look for “hard verses” with some additional videos besides the podcast. Check out my friends at Godseverlastingword.com.

    Face Toward Jerusalem, Luke 9:51

    Play Episode Listen Later Aug 2, 2016 24:35


      Hard Verses Episode 14: Face toward Jerusalem, Luke 9:51 Hard verses is a podcast not afraid to take on the harder verses in the Bible.  Each week we find difficult verses in God's word. Wanted to give props to Dr. Ed Gallagher for directing my mind to the verse because of his class. Limited Resource Dynamics in Ancient Israel. The Problem. Luk 9:51  And it came to pass, when the time was come that he should be received up, he stedfastly set his face to go to Jerusalem, Luk 9:52  And sent messengers before his face: and they went, and entered into a village of the Samaritans, to make ready for him. Luk 9:53  And they did not receive him, because his face was as though he would go to Jerusalem. What was going on here? What does it mean that He “stedfastly set His face to go to Jerusalem?” He sent messengers to Samaria and did not receive Him, because His face was though he would go to Jerusalem????  All Jews were supposed to go to Jerusalem for three feast days, so why are the Samaritans made at Him now? The Solution. Luk 9:51  And it came to pass, when the time was come that he should be received up, he stedfastly set his face to go to Jerusalem, Luk 9:52  And sent messengers before his face: and they went, and entered into a village of the Samaritans, to make ready for him. Luk 9:53  And they did not receive him, because his face was as though he would go to Jerusalem. Luk 9:54  And when his disciples James and John saw this, they said, Lord, wilt thou that we command fire to come down from heaven, and consume them, even as Elias did? What does it mean that He “steadfastly set His face to go to Jerusalem?” He resolved Himself to go to His death. They rejected Him because His face was though He would go to Jerusalem. The Samaritans knew where He was going.  The route (Josephus, Antiquities, 20.6.1) was that of one going to the Temple. The Samaritans had their own temple in Mt.Gerizim.  They thought that was the God ordained location.  They had Biblical reasons for that. Deu 11:29  And it shall come to pass, when the LORD thy God hath brought thee in unto the land whither thou goest to possess it, that thou shalt put the blessing upon mount Gerizim, and the curse upon mount Ebal. What caused even more division is the destruction of the temple at Mt. Gerizim. After the end of the Babylonian Captivity, a large schism between the Samaritans and Judaism developed, with the Samaritans, but not the Jews, regarding Mount Gerizim as theholy place chosen by God.[3] Subsequently, in the Persian Period, the Samaritans built a temple there probably in the middle of 5th century BCE.,[11] arguing that this was the real location of the Israelite temple which had been destroyed by Nebuchadnezzar,[26] The religious tension between the Jews and the Samaritans led to the temple on Gerizim being destroyed by either John Hyrcanus in the 2nd century BCE (according to Josephus)[27] or by Simeon the Just (according to the Talmud). The date of the Samaritan temple destruction, the 21st of Kislev, became a holiday for the Jews during which it is forbidden to eulogize the dead.[28] However, the mountain evidently continued to be the holy place of the Samaritans, as it is mentioned as such by the Gospel of John[29] and coins produced by a Roman mint situated in Nablus included within their design a depiction of the temple; surviving coins from this mint, dated to 138–161 CE, show a huge temple complex, statues, and a substantive staircase leading from Nablus to the temple itself.[30] (wiki page—sorry scholars). Joh 4:19  The woman saith unto him, Sir, I perceive that thou art a prophet. Joh 4:20  Our fathers worshipped in this mountain; and ye say, that in Jerusalem is the place where men ought to worship. Joh 4:21  Jesus saith unto her, Woman, believe me, the hour cometh, when ye shall neither in this mountain, nor yet at Jerusalem, worship the Father. Joh 4:22  Ye worship ye know not what: we know what we worship: for salvation is of the Jews. Joh 4:23  But the hour cometh, and now is, when the true worshippers shall worship the Father in spirit and in truth: for the Father seeketh such to worship him. Joh 4:24  God is a Spirit: and they that worship him must worship him in spirit and in truth. There was a true debate about where the temple should be. Jesus set her straight and told her how it was going to be. Back to the original questions. What does it mean that He “stedfastly set His face to go to Jerusalem?” He sent messengers to Samaria and did not receive Him, because His face was though he would go to Jerusalem???? All Jews were supposed to go to Jerusalem for three feast days, so why are the Samaritans made at Him now?   The lesson for us today. As we learn the word are we willing to change? Tools I Used Barnes, Pulpit, Josephus, TSK, Wikipedia   Call to action. Subscribe and rate. You can also write a review. (Itunes). Check out the new Bible Study.  “The Bible Genius Bible Study” now available for free on preacherbryan.com There are also the podcasts available on Youtube, look for “hard verses” with some additional videos besides the podcast. Check out my friends at Godseverlastingword.com.

    Mystery of the Kingdom

    Play Episode Listen Later Jul 27, 2016 26:34


      Hard Verses Episode 13: Mystery made known, Wanting to thank all those who have shared. It is working.  Wanted to thank Billy Hines for the suggestions. The Problem. Mat 13:11  He answered and said unto them, Because it is given unto you to know the mysteries of the kingdom of heaven, but to them it is not given. What mysteries were given? Do we know what the mysteries are? Are there still mysteries that we do not know in relation to this? The Solution. Mat 13:11  He answered and said unto them, Because it is given unto you to know the mysteries of the kingdom of heaven, but to them it is not given. Mat 13:12  For whosoever hath, to him shall be given, and he shall have more abundance: but whosoever hath not, from him shall be taken away even that he hath. Mat 13:13  Therefore speak I to them in parables: because they seeing see not; and hearing they hear not, neither do they understand. Mat 13:14  And in them is fulfilled the prophecy of Esaias, which saith, By hearing ye shall hear, and shall not understand; and seeing ye shall see, and shall not perceive: Mat 13:15  For this people's heart is waxed gross, and their ears are dull of hearing, and their eyes they have closed; lest at any time they should see with their eyes, and hear with their ears, and should understand with their heart, and should be converted, and I should heal them. Mat 13:16  But blessed are your eyes, for they see: and your ears, for they hear. Mat 13:17  For verily I say unto you, That many prophets and righteous men have desired to see those things which ye see, and have not seen them; and to hear those things which ye hear, and have not heard them. There are two groups; Those “given to know the mysteries.” Those not given. Of these groups we can surmise that the disciples were given to know. Of the group would be the religious elites such as the Pharisees and Scribes. We see also that they would not understand because they had a spiritual condition that prevented them from seeing and understanding.   What is the mystery that has been revealed? The mystery has been a long standing mystery. Rom 16:25  Now to him that is of power to stablish you according to my gospel, and the preaching of Jesus Christ, according to the revelation of the mystery, which was kept secret since the world began, 1Co 2:7  But we speak the wisdom of God in a mystery, even the hidden wisdom, which God ordained before the world unto our glory: Considered the prophecies from the Old Testament. Gen 3:15  And I will put enmity between thee and the woman, and between thy seed and her seed; it shall bruise thy head, and thou shalt bruise his heel. Isa 7:14  Therefore the Lord himself shall give you a sign; Behold, a virgin shall conceive, and bear a son, and shall call his name Immanuel. Didn’t prophets and heavenly beings know what was going on? Mat 13:17  For verily I say unto you, That many prophets and righteous men have desired to see those things which ye see, and have not seen them; and to hear those things which ye hear, and have not heard them. 1Pe 1:12  Unto whom it was revealed, that not unto themselves, but unto us they did minister the things, which are now reported unto you by them that have preached the gospel unto you with the Holy Ghost sent down from heaven; which things the angels desire to look into. Part of the mystery had to do with the Gentiles. Rom 11:25  For I would not, brethren, that ye should be ignorant of this mystery, lest ye should be wise in your own conceits; that blindness in part is happened to Israel, until the fulness of the Gentiles be come in. In the first century they were preaching about the revealed mystery. Mar 4:11  And he said unto them, Unto you it is given to know the mystery of the kingdom of God: but unto them that are without, all these things are done in parables: Eph 1:9  Having made known unto us the mystery of his will, according to his good pleasure which he hath purposed in himself: Eph 3:8  Unto me, who am less than the least of all saints, is this grace given, that I should preach among the Gentiles the unsearchable riches of Christ; Eph 3:9  And to make all men see what is the fellowship of the mystery, which from the beginning of the world hath been hid in God, who created all things by Jesus Christ: Col 1:26  Even the mystery which hath been hid from ages and from generations, but now is made manifest to his saints: Col 1:27  To whom God would make known what is the riches of the glory of this mystery among the Gentiles; which is Christ in you, the hope of glory: So how was it revealed?  Did they figure it out? No. It was revealed. By revelation of God. Eph 3:3  How that by revelation he made known unto me the mystery; (as I wrote afore in few words, Eph 3:4  Whereby, when ye read, ye may understand my knowledge in the mystery of Christ) Eph 3:5  Which in other ages was not made known unto the sons of men, as it is now revealed unto his holy apostles and prophets by the Spirit; Eph 3:6  That the Gentiles should be fellowheirs, and of the same body, and partakers of his promise in Christ by the gospel: Rom 16:25  Now to him that is of power to stablish you according to my gospel, and the preaching of Jesus Christ, according to the revelation of the mystery, which was kept secret since the world began, Rom 16:26  But now is made manifest, and by the scriptures of the prophets, according to the commandment of the everlasting God, made known to all nations for the obedience of faith: By the end of the book (the Bible) this had been fully revealed. Jud 1:3  Beloved, when I gave all diligence to write unto you of the common salvation, it was needful for me to write unto you, and exhort you that ye should earnestly contend for the faith which was once delivered unto the saints. There were already some trying to change the message. The church is supposed to teach and preach the whole gospel. Joh 15:15  Henceforth I call you not servants; for the servant knoweth not what his lord doeth: but I have called you friends; for all things that I have heard of my Father I have made known unto you. Mat 28:18  And Jesus came and spake unto them, saying, All power is given unto me in heaven and in earth. Mat 28:19  Go ye therefore, and teach all nations, baptizing them in the name of the Father, and of the Son, and of the Holy Ghost: Mat 28:20  Teaching them to observe all things whatsoever I have commanded you: and, lo, I am with you alway, even unto the end of the world. Amen. Act 20:26  Wherefore I take you to record this day, that I am pure from the blood of all men. Act 20:27  For I have not shunned to declare unto you all the counsel of God. The lesson for us today. The mystery is revealed. What are we going to do with this revealed message? Are we teaching the entire gospel or some abbreviated version? Tools I Used TSK   Call to action. Iheartradio is now carrying my show. Subscribe.

    Marry Only in the Lord; 1 Cor. 7:39

    Play Episode Listen Later Jul 19, 2016 17:30


      Hard Verses Episode 12: Marry only in the Lord; 1 Cor. 7:39 Wanted to thank everyone for sharing and all the positive response.  Wanted to thank everyone for all the new suggestions. Wanted to thank Larry White for the suggestion. The Problem. 1Co 7:39  The wife is bound by the law as long as her husband liveth; but if her husband be dead, she is at liberty to be married to whom she will; only in the Lord. What does it mean to be married to whom she will; only in the Lord? Can she only marry another Christian man? Can she only marry in a Christian manner? The Solution. The issue is what does “in the Lord” mean and then modify? Earlier “in the Lord” is apparent to mean a Christian in Paul’s other writings. Rom 16:11  Salute Herodion my kinsman. Greet them that be of the household of Narcissus, which are in the Lord.                  The NIV actually translates this passage this way. 39 A woman is bound to her husband as long as he lives. But if her husband dies, she is free to marry anyone she wishes, but he must belong to the Lord. Looking at other verses that may have to do with this, like being unequally yoked. 2Co 6:14  Be ye not unequally yoked together with unbelievers: for what fellowship hath righteousness with unrighteousness? and what communion hath light with darkness? 2Co 6:15  And what concord hath Christ with Belial? or what part hath he that believeth with an infidel? 2Co 6:16  And what agreement hath the temple of God with idols? for ye are the temple of the living God; as God hath said, I will dwell in them, and walk in them; and I will be their God, and they shall be my people. 2Co 6:17  Wherefore come out from among them, and be ye separate, saith the Lord, and touch not the unclean thing; and I will receive you,                 Is this discussing marriage?  Not specifically. The Greek does not seem to indicate this. I had two peer reviewed articles that I received from a search and neither dealt with the issue of verse 39. I went through my commentaries and almost all said that the man has to be a Christian without explaining why. Did a google search and came up with several church of Christ websites and one Jehovah Witness site (which did not explain the verse, just made an assertion). The issue is that if this forbidding of marriage to unbelievers is to be taken from this verse 39 then it would be the only verse forbidding this. There is also the issue of how to rectify the situation if this is a sinful act.  Will the woman now need to divorce? According to Matthew 19:9, there is only one reason for divorce. Mat 19:9  And I say unto you, Whosoever shall put away his wife, except it be for fornication, and shall marry another, committeth adultery: and whoso marrieth her which is put away doth commit adultery.                 Prior verses speak to woman staying married to their unbelieving husbands so that they may be converted. All this would be to point to the “in the Lord” as modifying the act of marrying. The woman would have to be married in a scriptural way.  They would not need to be married to someone with several wives.  They would also not need to marry someone that would require them to go to Pagan worship. This seems to be the way this is modified.  When looking at a very similar passage using “in the Lord,” The modified object is a little easier to spot. Eph 6:1  Children, obey your parents in the Lord: for this is right. Is “in the Lord” modifying Children, parents or the act of obeying? It seems obvious that this means the obedience is to be complete unless it requires a child to go against God’s law.   There is one other argument. This passage only applies because of the upcoming, eminent distress. This would mean that this verse is no longer applicable to us today.  Because there will be massive heart ache one needs to find a Christian spouse.  I personally don’t see this view.  That would make sense to those debating whether to marry or not.  Since taking a spouse is an added burden in a time of hardship. The lesson for us today. We need to be living our lives “in the Lord.” Tools I Used http://lavistachurchofchrist.org/LVarticles/OnlyInTheLord.html http://westpalmbeachchurchofchrist.com/diff_quest/only_in_the_lord.html https://www.christiancourier.com/articles/313-should-a-christian-marry-outside-the-faith e-sword Modules Barnes, Burton Coffman, Clarke, JFB.   Call to action. Youtube is up.  “Hard Verses.” Subscribe. If you have an Iphone or ipad, then you have Itunes.  Go to ITunes to the top right to the search box.  What is a search box?  It is a white rectangle with a magnifying glass on the left and a powder blue border around the box.  Put “Hard Verses” in the search. Hit enter.  Scroll down to the bottom of the page until to see the podcast section.  I should be the number one square picture. It has a picture of me with my parrot coffee cup.  Click on that. That is my show page. Underneath my picture is my subscribe button.  Hit that and all your dreams will come true.

    Calling on the Name of the Lord

    Play Episode Listen Later Jul 12, 2016 25:24


      Hard Verses Episode 11: Calling on the Name of the Lord.  Romans 10   Wanted to thank Billy Hines for the suggestion of today’s verses.  Also wanted to thank all of those that shared the episode last week.  I saw who you were.  Thank you so much.   The Problem. Romans 10 is pretty clear that we are to call upon the name of the Lord. Rom 10:8  But what saith it? The word is nigh thee, even in thy mouth, and in thy heart: that is, the word of faith, which we preach; Rom 10:9  That if thou shalt confess with thy mouth the Lord Jesus, and shalt believe in thine heart that God hath raised him from the dead, thou shalt be saved. Rom 10:10  For with the heart man believeth unto righteousness; and with the mouth confession is made unto salvation. Rom 10:11  For the scripture saith, Whosoever believeth on him shall not be ashamed. Rom 10:12  For there is no difference between the Jew and the Greek: for the same Lord over all is rich unto all that call upon him. Rom 10:13  For whosoever shall call upon the name of the Lord shall be saved. After someone gives this verse, what do they tell people to do to call upon the name of the Lord? Many of the answers given beg the question. Can we actually find someone in the Bible that calls upon the name of the Lord? Sound Transition New Commercial Sound Transition The Solution. What is the word in question? To call upon ἐπικαλέομαι epikaleomai ep-ee-kal-eh'-om-ahee Middle voice from G1909 and G2564; to entitle; by implication to invoke (for aid, worship, testimony, decision, etc.): - appeal (unto), call (on, upon), surname. Strong’s Concordance   Romans was written according to some winter of A.D. 56-57.   Rom 10:8  But what saith it? The word is nigh thee, even in thy mouth, and in thy heart: that is, the word of faith, which we preach; Rom 10:9  That if thou shalt confess with thy mouth the Lord Jesus, and shalt believe in thine heart that God hath raised him from the dead, thou shalt be saved. Rom 10:10  For with the heart man believeth unto righteousness; and with the mouth confession is made unto salvation. Rom 10:11  For the scripture saith, Whosoever believeth on him shall not be ashamed. Rom 10:12  For there is no difference between the Jew and the Greek: for the same Lord over all is rich unto all that call upon him. Rom 10:13  For whosoever shall call upon the name of the Lord shall be saved.   This was not first time that this was preached. We see it in Acts 2:21. Act 2:21  And it shall come to pass, that whosoever shall call on the name of the Lord shall be saved. This was said around AD 30.  This would be after the resurrection of Christ and His ascension into heaven and before the conversion of Saul of Tarsus.  Because the church was old enough to be persecuted by said Saul. And this was quoted in Joel. Joe 2:32  And it shall come to pass, that whosoever shall call on the name of the LORD shall be delivered: for in mount Zion and in Jerusalem shall be deliverance, as the LORD hath said, and in the remnant whom the LORD shall call.                 Peter’s message and Paul’s message should be same.  The Holy Spirit would have directed to give the same message from God.  They did not get to change the words to suit themselves  2Ti 3:16  All scripture is given by inspiration of God, and is profitable for doctrine, for reproof, for correction, for instruction in righteousness:   1Co 2:1  And I, brethren, when I came to you, came not with excellency of speech or of wisdom, declaring unto you the testimony of God. 1Co 2:2  For I determined not to know any thing among you, save Jesus Christ, and him crucified. 1Co 2:3  And I was with you in weakness, and in fear, and in much trembling. 1Co 2:4  And my speech and my preaching was not with enticing words of man's wisdom, but in demonstration of the Spirit and of power: 1Co 2:5  That your faith should not stand in the wisdom of men, but in the power of God.   What did Peter and the other apostles do after preaching to call upon the name of the Lord? Act 2:36  Therefore let all the house of Israel know assuredly, that God hath made that same Jesus, whom ye have crucified, both Lord and Christ. Act 2:37  Now when they heard this, they were pricked in their heart, and said unto Peter and to the rest of the apostles, Men and brethren, what shall we do? Act 2:38  Then Peter said unto them, Repent, and be baptized every one of you in the name of Jesus Christ for the remission of sins, and ye shall receive the gift of the Holy Ghost.   What about Paul? He wrote Roman’s ten, shouldn’t we have a record.   Act 9:17  And Ananias went his way, and entered into the house; and putting his hands on him said, Brother Saul, the Lord, even Jesus, that appeared unto thee in the way as thou camest, hath sent me, that thou mightest receive thy sight, and be filled with the Holy Ghost. Act 9:18  And immediately there fell from his eyes as it had been scales: and he received sight forthwith, and arose, and was baptized.   Act 22:6  And it came to pass, that, as I made my journey, and was come nigh unto Damascus about noon, suddenly there shone from heaven a great light round about me. Act 22:7  And I fell unto the ground, and heard a voice saying unto me, Saul, Saul, why persecutest thou me? Act 22:8  And I answered, Who art thou, Lord? And he said unto me, I am Jesus of Nazareth, whom thou persecutest. Act 22:9  And they that were with me saw indeed the light, and were afraid; but they heard not the voice of him that spake to me. Act 22:10  And I said, What shall I do, Lord? And the Lord said unto me, Arise, and go into Damascus; and there it shall be told thee of all things which are appointed for thee to do. Act 22:11  And when I could not see for the glory of that light, being led by the hand of them that were with me, I came into Damascus. Act 22:12  And one Ananias, a devout man according to the law, having a good report of all the Jews which dwelt there, Act 22:13  Came unto me, and stood, and said unto me, Brother Saul, receive thy sight. And the same hour I looked up upon him. Act 22:14  And he said, The God of our fathers hath chosen thee, that thou shouldest know his will, and see that Just One, and shouldest hear the voice of his mouth. Act 22:15  For thou shalt be his witness unto all men of what thou hast seen and heard. Act 22:16  And now why tarriest thou? arise, and be baptized, and wash away thy sins, calling on the name of the Lord. I thought Paul taught that it was just the calling on the name in chapter 10 of Romans? Had everyone read up to chapter 10 it would make more sense. Consider chapter six. Rom 6:3  Know ye not, that so many of us as were baptized into Jesus Christ were baptized into his death? Rom 6:4  Therefore we are buried with him by baptism into death: that like as Christ was raised up from the dead by the glory of the Father, even so we also should walk in newness of life. Rom 6:5  For if we have been planted together in the likeness of his death, we shall be also in the likeness of his resurrection: That is just Romans, correct? No. Col 2:12  Buried with him in baptism, wherein also ye are risen with him through the faith of the operation of God, who hath raised him from the dead. Gal 3:27  For as many of you as have been baptized into Christ have put on Christ. Yes, but there is nowhere that the Bible states that baptism saves you, right? 1Pe 3:21  The like figure whereunto even baptism doth also now save us (not the putting away of the filth of the flesh, but the answer of a good conscience toward God,) by the resurrection of Jesus Christ: Jesus never talked of baptism being necessary for salvation, right? Mar 16:15  And he said unto them, Go ye into all the world, and preach the gospel to every creature. Mar 16:16  He that believeth and is baptized shall be saved; but he that believeth not shall be damned. The lesson for us today. The story of the hammer drill.  You are doing it all wrong. Are we doing it all wrong with God’s plan? Tools I Used Treasury of Scriptural knowledge. Call to action. You have the week off unless you think one of your friends and family need to understand this better.

    Blaspheming the Holy Spirit

    Play Episode Listen Later Jul 5, 2016 32:16


      Please, Please, Please share on the social networks.  Thank you so much for the support. Hard Verses Episode 10: Blaspheming the Holy Spirit   Want to thank Ted Burleson.  About podcast and the class.   The Problem. Luk 12:10  And whosoever shall speak a word against the Son of man, it shall be forgiven him: but unto him that blasphemeth against the Holy Ghost it shall not be forgiven. How is it that speaking against the Son of God is forgivable but speaking against the Holy Ghost shall not be forgiven? If you speak against the Holy Ghost as a child or drunk or out of your mind would this mean you just give up and live like you want because you are eternally lost, no matter what you do? The Solution. Let’s look at the parallels. Luk 12:8  Also I say unto you, Whosoever shall confess me before men, him shall the Son of man also confess before the angels of God: Luk 12:9  But he that denieth me before men shall be denied before the angels of God. Luk 12:10  And whosoever shall speak a word against the Son of man, it shall be forgiven him: but unto him that blasphemeth against the Holy Ghost it shall not be forgiven. Luk 12:11  And when they bring you unto the synagogues, and unto magistrates, and powers, take ye no thought how or what thing ye shall answer, or what ye shall say: Luk 12:12  For the Holy Ghost shall teach you in the same hour what ye ought to say. Mat 12:22  Then was brought unto him one possessed with a devil, blind, and dumb: and he healed him, insomuch that the blind and dumb both spake and saw. Mat 12:23  And all the people were amazed, and said, Is not this the son of David? Mat 12:24  But when the Pharisees heard it, they said, This fellow doth not cast out devils, but by Beelzebub the prince of the devils.Mat 12:25  And Jesus knew their thoughts, and said unto them, Every kingdom divided against itself is brought to desolation; and every city or house divided against itself shall not stand:Mat 12:26  And if Satan cast out Satan, he is divided against himself; how shall then his kingdom stand? Mat 12:27  And if I by Beelzebub cast out devils, by whom do your children cast them out? therefore they shall be your judges. Mat 12:28  But if I cast out devils by the Spirit of God, then the kingdom of God is come unto you. Mat 12:29  Or else how can one enter into a strong man's house, and spoil his goods, except he first bind the strong man? and then he will spoil his house. Mat 12:30  He that is not with me is against me; and he that gathereth not with me scattereth abroad. Mat 12:31  Wherefore I say unto you, All manner of sin and blasphemy shall be forgiven unto men: but the blasphemy against the Holy Ghost shall not be forgiven unto men. Mat 12:32  And whosoever speaketh a word against the Son of man, it shall be forgiven him: but whosoever speaketh against the Holy Ghost, it shall not be forgiven him, neither in this world, neither in the world to come. Mat 12:33  Either make the tree good, and his fruit good; or else make the tree corrupt, and his fruit corrupt: for the tree is known by his fruit.Mat 12:34  O generation of vipers, how can ye, being evil, speak good things? for out of the abundance of the heart the mouth speaketh. Mat 12:35  A good man out of the good treasure of the heart bringeth forth good things: and an evil man out of the evil treasure bringeth forth evil things. Mat 12:36  But I say unto you, That every idle word that men shall speak, they shall give account thereof in the day of judgment. Mat 12:37  For by thy words thou shalt be justified, and by thy words thou shalt be condemned. Mar 3:22  And the scribes which came down from Jerusalem said, He hath Beelzebub, and by the prince of the devils casteth he out devils. Mar 3:23  And he called them unto him, and said unto them in parables, How can Satan cast out Satan? Mar 3:24  And if a kingdom be divided against itself, that kingdom cannot stand.Mar 3:25  And if a house be divided against itself, that house cannot stand. Mar 3:26  And if Satan rise up against himself, and be divided, he cannot stand, but hath an end. Mar 3:27  No man can enter into a strong man's house, and spoil his goods, except he will first bind the strong man; and then he will spoil his house. Mar 3:28  Verily I say unto you, All sins shall be forgiven unto the sons of men, and blasphemies wherewith soever they shall blaspheme: Mar 3:29  But he that shall blaspheme against the Holy Ghost hath never forgiveness, but is in danger of eternal damnation: Mar 3:30  Because they said, He hath an unclean spirit. What does it even mean to blaspheme? Blasphemeo- to vilify, specifically to speak impiously, defame, rail on, revile, speak evil.   How were they to blaspheme Jesus and still receive forgiveness? Consider Acts chapter 2. Act 2:22  Ye men of Israel, hear these words; Jesus of Nazareth, a man approved of God among you by miracles and wonders and signs, which God did by him in the midst of you, as ye yourselves also know: Act 2:23  Him, being delivered by the determinate counsel and foreknowledge of God, ye have taken, and by wicked hands have crucified and slain: Act 2:24  Whom God hath raised up, having loosed the pains of death: because it was not possible that he should be holden of it. Act 2:36  Therefore let all the house of Israel know assuredly, that God hath made that same Jesus, whom ye have crucified, both Lord and Christ. Act 2:37  Now when they heard this, they were pricked in their heart, and said unto Peter and to the rest of the apostles, Men and brethren, what shall we do? Act 2:38  Then Peter said unto them, Repent, and be baptized every one of you in the name of Jesus Christ for the remission of sins, and ye shall receive the gift of the Holy Ghost. Act 2:40  And with many other words did he testify and exhort, saying, Save yourselves from this untoward generation. Act 2:41  Then they that gladly received his word were baptized: and the same day there were added unto them about three thousand souls.   How did they revile or speak evil of the Holy Spirit? They had attributed the power of the Holy Spirit with the power of Satan. Are we saying that those Pharisees saying these things are eternally lost? Consider the tense of the word blasphemeth.  There is a continual action.  If every time you see a miracle of God and attribute that to the work of Satan, there is not much hope for you.  Notice Mark 3:29  But he that shall blaspheme against the Holy Ghost hath never forgiveness, but is in danger of eternal damnation: Mar 3:30  Because they said, He hath an unclean spirit. Consider also that the words of the Holy Spirit that will be spoken through the apostles after the resurrection and ascension.  These would be the words of the New Testament, the new covenant.  The Christian age.  The final age on earth.  You miss this, that is it. Notice-- Luk 12:10  And whosoever shall speak a word against the Son of man, it shall be forgiven him: but unto him that blasphemeth against the Holy Ghost it shall not be forgiven. Luk 12:11  And when they bring you unto the synagogues, and unto magistrates, and powers, take ye no thought how or what thing ye shall answer, or what ye shall say: Luk 12:12  For the Holy Ghost shall teach you in the same hour what ye ought to say. Consider for us today, Luke talks about confessing Jesus before men and then mentioning the blaspheming of the Holy Spirit. Consider who these people were that had this message. They were Pharisees, Scribes, Doctors of the law (lawyers).  They knew the law of Moses.  They knew the prophesies about the Messiah, yet they rejected the Messiah outright because of their reprobate heart. Is our heart so lost that we do not see the clear teaching of the gospel? To sum up the most prevalent from the evidence about what the blasphemy of the Holy Ghost is, let me sum it up in points. This was a spoken sin or sin of the tongue. Mark clearly states that Jesus said this because they said , “He has an unclean spirit.” We see the Pharisees speaking ill of the power of God performing these miracles. Consider Paul who stated that he had been formerly been “a blasphemer, a persecutor…” 1Ti 1:13  Who was before a blasphemer, and a persecutor, and injurious: but I obtained mercy, because I did it ignorantly in unbelief.                 Notice what Paul’s reaction was when faced with a miraculous event.  “This is by the power of Satan.” NO! Act 9:1  And Saul, yet breathing out threatenings and slaughter against the disciples of the Lord, went unto the high priest, Act 9:2  And desired of him letters to Damascus to the synagogues, that if he found any of this way, whether they were men or women, he might bring them bound unto Jerusalem. Act 9:3  And as he journeyed, he came near Damascus: and suddenly there shined round about him a light from heaven: Act 9:4  And he fell to the earth, and heard a voice saying unto him, Saul, Saul, why persecutest thou me? Act 9:5  And he said, Who art thou, Lord? And the Lord said, I am Jesus whom thou persecutest: it is hard for thee to kick against the pricks. Act 9:6  And he trembling and astonished said, Lord, what wilt thou have me to do? And the Lord said unto him, Arise, and go into the city, and it shall be told thee what thou must do. Act 9:7  And the men which journeyed with him stood speechless, hearing a voice, but seeing no man. Act 9:8  And Saul arose from the earth; and when his eyes were opened, he saw no man: but they led him by the hand, and brought him into Damascus. Can we commit this sin today? Consider these verses. Consider the time frame. 1Jn 1:7  But if we walk in the light, as he is in the light, we have fellowship one with another, and the blood of Jesus Christ his Son cleanseth us from all sin. 1Jn 1:8  If we say that we have no sin, we deceive ourselves, and the truth is not in us. 1Jn 1:9  If we confess our sins, he is faithful and just to forgive us our sins, and to cleanse us from all unrighteousness. Was this sin ever mentioned in the New Testament church?  No. Now we have the gospel that was set motion because of the blood that was shed on Calvary.     The lesson for us today. There is a connection for us today. Is the word confirmed to us today and we are still rejecting it? Heb 6:4  For it is impossible for those who were once enlightened, and have tasted of the heavenly gift, and were made partakers of the Holy Ghost,Heb 6:5  And have tasted the good word of God, and the powers of the world to come,Heb 6:6  If they shall fall away, to renew them again unto repentance; seeing they crucify to themselves the Son of God afresh, and put him to an open shame. Do you understand what the Bible says and you simply are not doing it? You need to recognize what you are doing. Are we not familiar with the message of the Holy Spirit through the apostles and prophets in the New Testament?  We may be speaking against something that you should be adhering to.  My mom tells the story of her praying, “I take your name in vain.”   Tools I Used Student’s Pericope book (I have a copy but it has not been published or printed yet).  Let me know if you are interested in getting a copy.  My section is placed in the show notes below. Burton Coffman module in e-sword. Treasury of Scriptural Knowledge module in e-sword. http://apologeticspress.org/apcontent.aspx?category=11&article=1218#   Call to action. Share on social media, THIS EPISODE, PLEASE, PLEASE, PLEASE (Imagine a little child at Walmart screaming at you).  

    The Land of Nod

    Play Episode Listen Later Jun 28, 2016 13:12


      Hard Verses Episode 9: The Land of Nod   Wanted to take time to give a little thanks. Jake for loaning me his Zoom H4N to record. Wanted to thank Blue Microphones for sending us a replacement for the one with a hum in it. We have started a Bible study program at Flomaton entitled “Bible Genius” Making the weeks available online.   The Problem. The Land of Nod. Gen 4:13  And Cain said unto the LORD, My punishment is greater than I can bear. Gen 4:14  Behold, thou hast driven me out this day from the face of the earth; and from thy face shall I be hid; and I shall be a fugitive and a vagabond in the earth; and it shall come to pass, that every one that findeth me shall slay me. Gen 4:15  And the LORD said unto him, Therefore whosoever slayeth Cain, vengeance shall be taken on him sevenfold. And the LORD set a mark upon Cain, lest any finding him should kill him. Gen 4:16  And Cain went out from the presence of the LORD, and dwelt in the land of Nod, on the east of Eden. Gen 4:17  And Cain knew his wife; and she conceived, and bare Enoch: and he builded a city, and called the name of the city, after the name of his son, Enoch. Some questions should come to mind. What is the deal with the land of Nod? Was there already people there? If so, where did they come from (thought God only miraculously created Adam and Eve?) Where did Cain find him a woman? Maybe a personal story or two about others that have encountered this issue.   The Solution. What are we assuming that is making this difficult? That there was an established land called Nod that Cain went to. That Cain received his wife from this land. The Land of Nod- “Nod” means wandering. The land of Nod may have been founded way later.  (Prolepsis). Gen 13:1  And Abram went up out of Egypt, he, and his wife, and all that he had, and Lot with him, into the south. Gen 13:2  And Abram was very rich in cattle, in silver, and in gold. Gen 13:3  And he went on his journeys from the south even to Bethel, unto the place where his tent had been at the beginning, between Bethel and Hai; This is a problem because Bethel was not formed yet nor named.  It would be named by his now unborn grandson in Gen 28:19. Gen 28:18  And Jacob rose up early in the morning, and took the stone that he had put for his pillows, and set it up for a pillar, and poured oil upon the top of it. Gen 28:19  And he called the name of that place Bethel: but the name of that city was called Luz at the first. The land could have been named at a later date like Abram going to Bethel when it would be named by his grandson decades later.   The issue of a wife, solved because it was his sister.  Just as Abram had his sister as his wife.   The lesson for us today. Things allowed in the Old Testament many times are not good for today. Don’t kill your brother. Don’t marry your sister. Don’t take things out of the Bible that are not there.   Tools I Used http://apologeticspress.org/apcontent.aspx?category=6&article=696#   Call to action.   Go to Preacherbryan.com. Just click on the episode, press play. Check out the new Bible Genius bible study in part. Subscription Stitcher, Itunes, Google Play, I have added Facebook and Twitter publishing. AND YOUTUBE BABY. Get connected: Facebook- Bryan Cook Twitter- Preacherbryan Google +- Bryan Cook Snapchat but don’t know what to do with it.  

    Rahab Lied, Joshua 2

    Play Episode Listen Later Jun 21, 2016 25:42


      Hard Verses Episode 8: Rahab Lied (Joshua 3, Hebrews 11:31, James 2:25). Wanted to thank Derek Crump for the suggestion for this episode.   The Problem. Let me take you back.  The children are about to go into the Promised Land in the book of Joshua.  This time they will make it.  The spies went into Jericho and this woman, Rahab the harlot, lied and hid the spies to have them escape.  She is heralded as a woman of faith and acclaim by God’s inspired word (Heb. 11:31, James 2:25).  This means that God is tolerant of lying as long as it is for a greater good.  This is called “situational ethics.”  Could this be the case?  I thought that all liars have their place in the lake of fire (Rev. 21:8).  This sounds like a problem. The Solution. What exactly does it say? Jos_2:1  And Joshua the son of Nun sent out of Shittim two men to spy secretly, saying, Go view the land, even Jericho. And they went, and came into an harlot's house, named Rahab, and lodged there. Jos 2:2  And it was told the king of Jericho, saying, Behold, there came men in hither to night of the children of Israel to search out the country. Jos_2:3  And the king of Jericho sent unto Rahab, saying, Bring forth the men that are come to thee, which are entered into thine house: for they be come to search out all the country. Jos 2:4  And the woman took the two men, and hid them, and said thus, There came men unto me, but I wist not whence they were: Jos 2:5  And it came to pass about the time of shutting of the gate, when it was dark, that the men went out: whither the men went I wot not: pursue after them quickly; for ye shall overtake them. Jos 2:6  But she had brought them up to the roof of the house, and hid them with the stalks of flax, which she had laid in order upon the roof. Jos 2:7  And the men pursued after them the way to Jordan unto the fords: and as soon as they which pursued after them were gone out, they shut the gate. Jos 2:8  And before they were laid down, she came up unto them upon the roof; Jos 2:9  And she said unto the men, I know that the LORD hath given you the land, and that your terror is fallen upon us, and that all the inhabitants of the land faint because of you. Jos 2:10  For we have heard how the LORD dried up the water of the Red sea for you, when ye came out of Egypt; and what ye did unto the two kings of the Amorites, that were on the other side Jordan, Sihon and Og, whom ye utterly destroyed. Jos 2:11  And as soon as we had heard these things, our hearts did melt, neither did there remain any more courage in any man, because of you: for the LORD your God, he is God in heaven above, and in earth beneath. Jos 2:12  Now therefore, I pray you, swear unto me by the LORD, since I have shewed you kindness, that ye will also shew kindness unto my father's house, and give me a true token: Jos 2:13  And that ye will save alive my father, and my mother, and my brethren, and my sisters, and all that they have, and deliver our lives from death. Jos 2:14  And the men answered her, Our life for yours, if ye utter not this our business. And it shall be, when the LORD hath given us the land, that we will deal kindly and truly with thee. Jos 2:15  Then she let them down by a cord through the window: for her house was upon the town wall, and she dwelt upon the wall. Jos 2:16  And she said unto them, Get you to the mountain, lest the pursuers meet you; and hide yourselves there three days, until the pursuers be returned: and afterward may ye go your way. Jos 2:17  And the men said unto her, We will be blameless of this thine oath which thou hast made us swear. Jos_6:17  And the city shall be accursed, even it, and all that are therein, to the LORD: only Rahab the harlot shall live, she and all that are with her in the house, because she hid the messengers that we sent. Jos_6:23  And the young men that were spies went in, and brought out Rahab, and her father, and her mother, and her brethren, and all that she had; and they brought out all her kindred, and left them without the camp of Israel. Jos_6:25  And Joshua saved Rahab the harlot alive, and her father's household, and all that she had; and she dwelleth in Israel even unto this day; because she hid the messengers, which Joshua sent to spy out Jericho. What all did she do that was so notable? She hid the spies (against the king’s orders). She bold-faced lied about their whereabouts and fabricated a story. She proclaimed her belief in their Lord (He is God in heaven above and in earth beneath, Joshua 2:11). She helped the spies out of the city. She made them promise to remember her and her family when they came back. Heb_11:31  By faith the harlot Rahab perished not with them that believed not, when she had received the spies with peace. Her faith saved her. When she received the spies.   Jas_2:25  Likewise also was not Rahab the harlot justified by works, when she had received the messengers, and had sent them out another way? She was justified by works. When she received the spies and helped them escape. What is missing is a commendation for lying. Lying is a direct violation of God’s law, Old and New Testament (Ex. 20:16Lev. 19:11; Prov. 6:6-19, Eph. 4:25, Col. 3:9).[1] The spies never asked her to lie.  The spies probably did not even know that she had lied.[2] Consider her position in life as well.  Wayne Jackson states the following” The case of Rahab is an example of where God honored a person due to their obedient faith in spite of a personal character flaw. Consider the following facts about this incident. Rahab was from a pagan environment. Her concept of morality and her personal lifestyle (she was a harlot) needed considerable refining. In spite of this sordid background, she had developed a genuine faith in Jehovah (Josh. 2:9ff.). She referred to Him as “God in heaven above, and on earth beneath....” Accordingly, when the spies approached her, she was not “disobedient” as were the others of Jericho, who perished in the destruction of the city, for she, through faith in God and His promises, received the spies in peace (Heb. 11:31), hid them, and sent them out another way (Jas. 2:25) [1986, 22:23].[3] It is apparent that Rahab did not continue to lie nor was she continuing her profession. What ever happened to Rahab? Mat 1:4  And Aram begat Aminadab; and Aminadab begat Naasson; and Naasson begat Salmon; Mat 1:5  And Salmon begat Booz of Rachab; and Booz begat Obed of Ruth; and Obed begat Jesse; Mat 1:6  And Jesse begat David the king; and David the king begat Solomon of her that had been the wife of Urias;   Rth 4:18  Now these are the generations of Pharez: Pharez begat Hezron, Rth 4:19  And Hezron begat Ram, and Ram begat Amminadab, Rth 4:20  And Amminadab begat Nahshon, and Nahshon begat Salmon, Rth 4:21  And Salmon begat Boaz, and Boaz begat Obed, Rth 4:22  And Obed begat Jesse, and Jesse begat David. The lesson for us today. If we are looking in the Bible to find ways to sin, we need to stop. We can never be wrong by respecting the living God. We may not know everything the Bible says to do, but what we do know we need to do to the best of our ability. Tools I Used E-sword module “Treasury of Scriptural Knowledge” or TSK (it is free for download). Article used below (“Does the Story of Rahab Mean that God Condones Lying?” by Bert Thompson,Ph.D.: http://www.apologeticspress.org/APContent.aspx?category=6&article=733   Call to action.   Go to Preacherbryan.com. Just click on the episode, press play. Subscription Stitcher, Itunes, Google Play, I have added Facebook and Twitter publishing. Get connected: Facebook- Bryan Cook Twitter- Preacherbryan Google +- Bryan Cook Snapchat but don’t know what to do with it.   [1] List of scriptures taken from Apologetics Press, “Does the Story of Rahab Mean that God Condones Lying?” by Bert Thompson, Ph.D. [2] Ibid. [3] Ibid.

    Inscribed on the Vesture and Thigh, Rev. 19:16

    Play Episode Listen Later Jun 14, 2016 21:46


    Hard Verses Episode 7: Inscription on the vesture and thigh (Rev. 19:16).     Wanted to clarify from last week the 1 Peter 3:18-19 (He spoke to the spirits that were in prison was from Don Gary, not Jordan Gray. The Problem. Rev 19:16 “And he hath on his vesture and on his thigh a name written, KING OF KINGS, AND LORD OF LORDS.” Much of Revelation has difficulty in nailing down concrete situations and details because of the symbolic language used. This verse is no different. The book is coded up because otherwise it would not get delivered or cause problems for the messengers. The Solution. Rev 19:16 “And he hath on his vesture and on his thigh a name written, KING OF KINGS, AND LORD OF LORDS.” The main problem we are having is a lack of historical knowledge on this custom. For time sack, I will not get into the previous and post verses but will focus strictly on this. What is a vesture? What is up with the thigh? “Two images likewise of this man are seen in Ionia, on the way that leads from Ephesus to Phocaea, and from Sardis to Smyrna. The figure is five palms in height; in his right hand he holds a dart, in his left a bow, armed after the manner of the Egyptians and Ethiopians. On a line drawn across the breast, from one shoulder to the other, are these words, written in Egyptian hieroglyphics: Εγω τηνδε την χωρην ωμοισι τοισι εμοισι εκτησαμην· ‘I obtained this country by these my shoulders;’“ i.e., by my own power. Heroditus, Euterpe speaking of the actions of Sesostris. In the Etruria Regalis of Dempster, in the appendix at the end of vol. ii., there is a beautiful female figure of brass, about twelve inches high, the hair gracefully plaited, and the head adorned with a diadem. She has a tunic without sleeves, and over that a sort of pallium. On the outside of the right thigh, close to the tunic, and probably on it, in the original, is an inscription in Etruscan characters. As these kinds of inscriptions on the thigh, the garments, and different parts of the body, were in use among different nations, to express character, conduct, qualities, and conquests, we may rest assured that to them St. John alludes when he represents our sovereign Lord with an inscription upon his vesture and upon his thigh; and had we not found it a custom among other nations, we should have been at a loss to account for its introduction and meaning here. Adam Clark Some suggestions from Pulpit Commentary below. (1) The name, written at length, is written partly upon the vesture and partly upon the thigh itself, where the garment would (in an equestrian figure) fall away from the thigh (Alford). (2) The name is written on the vesture, even (kai) on that part of it which covers the thigh (De Wette, Dusterdieck, Hengstenberg). (3) On the thigh, as the place where the sword usually hangs. (4) A reference to the custom of engraving the name of the artist upon the thigh of a statue (Cic., "Verr.," 4:43; see Wetstein). KING OF KINGS, AND LORD OF LORDS. As in Rev_17:14 (but inverted), where, as here, it portrays the victorious career of Christ over the "kings of the earth." - end of Pulpit Commentary. Studies in the Book of Revelation by Robert R. Taylor, Jr. An excerpt about Rev. 19:16. “Verse 16- On the garment and thigh of the Warrior King was an inscribed name- KING OF KINGS AND LORD OF LORDS. In the Gospel Advocate Adult Quarterly for the Winter of 1978 we have this fine summary of Revelation 19:16: His Lordship and sovereignty are emphasized here. All are subjects to his power. Kings of the earth, however powerful, are beneath him; lords, however, great, must ultimately answer to him. It is said that in ancient times and in Oriental lands it was not uncommon for men of great power and prominence to have their names inscribed on their garments so that they might be quickly recognized. We must not assume that such was literally necessary for the Lord to do in order that he might properly be recognized; the statement is a figurative one and the reference symbolic; it simply suggests the supreme power and unquestioned sovereignty of the Lord when he triumphs over his enemies (p. 54).   What about this name? continued. the number seven finds a place in the scheme of the Apocalypse so often that the following fact is well worth noting: leaving out the "and," by which St. John tells us to add the two titles into one name, the combination "King of Kings, Lord of Lords," in Aramaic [font unavailable to render] adds up to 777.[1]   The lesson for us today. Do we recognize Jesus as Lord of lords and King of kings? Do we recognize His commandments as law or are they just suggestions? Love your enemy, Seek ye first, etc. just suggestions or commandments?   Call to action.   Through e-sword (free for download, free to use) I used the following modules: Adam Clarke Burton Coffman Pulpit Commentary   Website has been updated preacherbryan.com to have a forum and the podcast front and center. Just click on the episode, press play. Subscription Stitcher, Itunes, Google Play Get connected: Facebook- Bryan Cook Twitter- Preacherbryan Google +- Bryan Cook   [1] KING OF KINGS, LORD OF LORDS (APOC. 19:16) Patrick W. Skehan.

    Preaching to the Spirits in Prison, 1 Peter 3:19

    Play Episode Listen Later Jun 7, 2016 34:24


      Hard Verses Episode 6: He Preached to the Spirits in Prison (1 Peter 3:19). Wanted to thank Sam Cook for the Intro work, Jordan Gray (actually Don Garey, sorry) for the suggestion and Alan Caudle for the legal disclaimer at the end. The Problem. For Christ also hath once suffered for sins, the just for the unjust, that he might bring us to God, being put to death in the flesh, but quickened by the Spirit: By which also he went and preached unto the spirits in prison; Which sometime were disobedient, when once the longsuffering of God waited in the days of Noah, while the ark was a preparing, wherein few, that is, eight souls were saved by water.  1 Peter 3:18-20 KJV Who? Who is the “he” in “by which also he went and preached unto the spirits in prison?”   What? Who or what are the spirits in prison? What did He preach to the spirits in prison? When? When did He preach to the spirits in prison? Where? Where were the spirits in prison? Where did Jesus preach to the spirits? Why? Why did He preach to the spirits in prison? Sound Transition The Solution. The whole purpose of this section is to encourage the brethren about the suffering that was on them. I will give you two “safe”[1] ways to interpret this. Jesus through the preaching of the Apostles and others presented the gospel to those in the same condition as those in Noah’s day. They would be captive and dead in their sin but now multitudes will come to Christ.   Eph 2:17. And came and preached peace to you which were afar off, and to them that were nigh.   –Jesus did not personally preach to the Gentiles but had his messengers bring the message. Could it be that Noah’s message to the lost was through the Holy Spirit, (the “in which” refers to the Spirit and not Christ) or that Christ would have been the one preaching through Noah? There are three major sets of beliefs with many subsets concerning this including our “safe” way to interpret this. A more recent view is what is stated by Pulpit Commentary. The preaching is in reference to men “alive after Pentecost to whom the gospel was preached by Christ through the apostles, men in a natural prison house of bandage to sin and Satan.”[2] An older theory that goes back to the third century AD, is that the spirits who are now in prison are “the disembodied souls of the people who perished in the Flood, and that the preincarnate Christ preached to them through Noah…”[3] The apparent oldest of the theories identifies spirits in prison as fallen angels. There are writings (unscriptural and uninspired) that indicate that this may have been the belief at the time of Peter’s writings. 1 Enoch predates not only Peter’s writings but even the ministry of Christ. There is additional support for this in that 2 Peter 2:4,5 and Jude 6 discuss the fate of angels and demons. There is descriptions of Christ being over spiritual beings. There are descriptions of angels and demons being reprimanded and subjugated. Spirit in these contemporary (inspired writings) is generally shown to describe the supernatural unless described additionally as human. Those early (uninspired) writers all point to the great transgression of the angels as occurring before the flood. (it may be common myth). According to Hiebert, this theory went into disfavor with the fourth century church. Back to the original questions. For Christ also hath once suffered for sins, the just for the unjust, that he might bring us to God, being put to death in the flesh, but quickened by the Spirit: By which also he went and preached unto the spirits in prison; Which sometime were disobedient, when once the longsuffering of God waited in the days of Noah, while the ark was a preparing, wherein few, that is, eight souls were saved by water.  1 Peter 3:18-20 KJV Who? Who is the “he” in “by which also he went and preached unto the spirits in prison?” Most would agree this is Christ even if pointing to the antecedent Spirit (the spirit of Christ).   What? Who or what are the spirits in prison? Disembodied souls of the lost at the time of Noah. (This does not go with other scriptures). The souls in that lost condition of those during the time of the apostles preaching. Angelic beings that were disobedient during the time prior to the ark. What did He preach to the spirits in prison? Disembodied souls lost at the time of Noah— He really could not preach the gospel because His resurrection was not complete. He would be giving them a second chance even though they rejected the message the entire century that Noah preached to them (not likely). The souls in a lost condition of sin. Preached the gospel to come out of that. They would be encouraged by this message (remember the purpose of this scripture). Only a few survived on the ark but many more are coming to know salvation through Christ. Angelic beings that were disobedient during the time prior to the ark. This would have been a proclamation of victory over them. Preaching is not always the gospel but can be a negative message. When? When did He preach to the spirits in prison? Disembodied souls lost at the time of Noah. He would have gone after dying on the cross and before or after His resurrection. (There is much confusion concerning the use of hell in the modern translations). To say that Christ traveled to the side of Gehenna (think of Lazarus and the rich man- the fiery side), would be to add to the word. Jesus stated that He would be with the thief on the cross in Paradise after death, not Gehenna (splitting hairs, maybe?). Where? Where were the spirits in prison? Where did Jesus preach to the spirits? Could the proclamation be made from heaven? Yes. Victory over death and the grave.  All power given unto Him over all things including spiritual.   Why? Why did He preach to the spirits in prison? Was He giving a second chance? To the already departed human and the disobedient angel, there is no second chance. To the men and women still alive, there was a second chance to not be like Noah’s generation that perished. Concerning the proclamation of victory over disobedient angels, this would have been in line with the proclamation that every knee will bow and tongue confess (Rom. 14:11; Phil. 2:10). This proclamation would also be encouraging to those currently suffering for Christ (remember the original intent of the section of scripture). The lesson for us today. We can draw encouragement from this scripture as well.  Are we suffering because we are doing God’s will? Keep in mind who is ultimately in charge. Maybe we need to be reminded of the entire population of the world drowning in a world-wide flood because they would not take the time to hear what God had to say. If we are lost and still in our sin then we are in bondage to something we have no way of getting out of ourselves.  We must heed the warning and obey the gospel.   Call to action. The links to the articles are here: http://apologeticspress.org/aparticle.aspx?cid=851 https://faculty.gordon.edu/hu/bi/ted_hildebrandt/ntesources/ntarticles/bsac-nt/hiebert-1peter3-pt2-bs.htm Through e-sword (free for download, free to use) I used the following modules: Burton Coffman   Pulpit Commentary   Website has been updated preacherbryan.com to have a forum and the podcast front and center. Just press play. Subscription are slowly being integrated. Let me know if you are not seeing Hard Verses on your favorite player. Please, don't be antisocial. Get connected: Facebook- Bryan Cook Twitter- Preacherbryan Google +- Bryan Cook   [1] Safe in that one could understand this without wandering from basic theological premises.  Other theories involve the use of more extravagant premises beyond basic theological tenets.  Could not think of another word to describe this (It is late when I am writing this). [2] The Suffering and Triumphant Christ: An Exposition of 1 Peter 3:18-22 by D. Edmond Hiebert.  His listing of the three theories is also used. [3] Ibid.

    The Graves Were Opened and the Saints Arose, Matthew 27:52-53

    Play Episode Listen Later May 31, 2016 27:33


    Hard Verses Episode 5: The Graves Were Opened and the Saints Arose (Matthew 27:52-53). Wanted to thank Miriam for the suggestion. She had posted on social media the suggestion.  Her post came up first. The Problem. Jesus, when he had cried again with a loud voice, yielded up the ghost. And, behold, the veil of the temple was rent in twain from the top to the bottom; and the earth did quake, and the rocks rent; And the graves were opened; and many bodies of the saints which slept arose, And came out of the graves after his resurrection, and went into the holy city, and appeared unto many. Now when the centurion, and they that were with him, watching Jesus, saw the earthquake, and those things that were done, they feared greatly, saying, truly this was the Son of God. (Matthew 27:50-54 KJV). At the time of Jesus’ death there are several things that occur. Darkness over the land. The veil of the temple was torn into. The earthquake. The rocks broke. The graves were opened. Many bodies of the dead saints arose. They came out of the graves after His resurrections. They then went into Jerusalem and appeared unto many. The centurion and those that were with him that were… Watching Jesus. Saw the earthquake. Saw other “things that were done. Feared greatly. Saying truly this was the Son of God. What makes this difficult? Were there really dead saints that arose? Were they Old Testament saints or those contemporary to Jesus that had recently died? The centurion would not have seen the rent in the Temple. The earthquake and the renting of the rocks would have been on Friday while the resurrection of the dead saints would have had to be after the resurrection of Jesus on Sunday since He is the first fruits of the resurrection of the dead (cf. 1 Cor. 15:20; Rev. 1:5; Acts 26:23; Col. 1:15; James 1:18). Is there any historical evidence of this occurrence outside of scripture? The Solution. There has to be more than just the witnessing of the earthquake and the renting rocks by the centurion.  “…when the centurion, and they that were with him, watching Jesus, saw the earthquake, and those things that were done…” What does this timeline look like? The centurion and those with him witnessed these events. What are these events that they saw? The darkness. The earthquakes. The rending of the rocks. The divine meekness of this crucified man. Heard His last words (Father forgive them). The raising of resuscitated saints (not likely). What would they not have seen? The renting of the temple veil. The raising of resurrected saints. Them coming to Jerusalem. Could this be a resuscitation instead of a resurrection? The historicity is good according to many that knew the eyewitnesses (taken from Geisler). The book itself (Matthew) claims to be an historical account. Many of the occurrences of Matthew are known to be historical from several sources even secular sources. The events were part of a section of scripture clearly historical in nature (the crucifixion, death, burial, etc.). Looking back at writers shortly after the Bible of “Church Fathers,” speak of Matthew’s account of the raising of the saints at the resurrection of Christ as an historical event (See Geisler article). The alleged confusion of early writers concerning this section of scripture, is not ambiguous as far as the historicity is concerned. There are those early writers that debate whether they rose at the death of Christ or at His resurrection. There was dispute as to what the centurion actually witnessed. None of the writers disputed whether it was historically accurate. The saints were probably those contemporary with Jesus or they would not have been recognized.     The lesson for us today. ???? What you listener should do next. The links to the articles are here (read it, it is excellent):    http://defendinginerrancy.com/early-fathers-resurrection-saints/ Website has been updated preacherbryan.com (Give it a look, it is ok). Subscription are slowly being integrated. Let me know if you are not seeing Hard Verses on your favorite player. Itunes, Stitcher, and Google Play Please, don't be antisocial. Get connected: Facebook- Bryan Cook Twitter- Preacherbryan Google +- Bryan Cook

    The Unjust Steward Part 2

    Play Episode Listen Later May 24, 2016 30:48


    Hey You!!! Download this file below.  You will need it. The Unjust Steward Digging Deeper into the Parable Hard Verses Episode 4: The Unjust Steward. (Luke 16:1-13) Part 2.     The Problem. And he said also unto his disciples, There was a certain rich man, which had a steward; and the same was accused unto him that he had wasted his goods. And he called him, and said unto him, How is it that I hear this of thee? give an account of thy stewardship; for thou mayest be no longer steward. Then the steward said within himself, What shall I do? for my lord taketh away from me the stewardship: I cannot dig; to beg I am ashamed.  I am resolved what to do, that, when I am put out of the stewardship, they may receive me into their houses. So he called every one of his lord's debtors unto him, and said unto the first, How much owest thou unto my lord?  And he said, An hundred measures of oil. And he said unto him, Take thy bill, and sit down quickly, and write fifty.  Then said he to another, And how much owest thou? And he said, An hundred measures of wheat. And he said unto him, Take thy bill, and write fourscore. And the lord commended the unjust steward, because he had done wisely: for the children of this world are in their generation wiser than the children of light.  And I say unto you, Make to yourselves friends of the mammon of unrighteousness; that, when ye fail, they may receive you into everlasting habitations. He that is faithful in that which is least is faithful also in much: and he that is unjust in the least is unjust also in much.  If therefore ye have not been faithful in the unrighteous mammon, who will commit to your trust the true riches?  And if ye have not been faithful in that which is another man's, who shall give you that which is your own?  No servant can serve two masters: for either he will hate the one, and love the other; or else he will hold to the one, and despise the other. Ye cannot serve God and mammon.                                                                                                                                                                 Luke 16:1-13 KJV I have actually had people say there is nothing you can get from this parable and that Jesus would not have said this. Whoever can come up with a solution to this parable can have a dozen people shoot it down with objections. Let us look at some of these problems. Why did the lord commend the unjust steward for giving away his product? Why did the lord commend the unjust steward while he was being dishonest and going behind his back? How are the children of the world wiser than the children of light? How could Jesus say to make friends of the unrighteous mammon and that this could produce everlasting habitation (Heaven)?   The Solution. We will be looking at the different elements of the parable in historical and cultural context (See attached PDF, “The Unjust Steward: Digging Deeper into the Parable.”)     The lesson for us today. We should be prudent and motivated to us money for an eternal purpose. We are to take care of the poor and less fortunate. We are to not let money come ahead of serving God.     Call to action. Download the pdf “The Unjust Steward.” This is a master’s level paper with peer reviewed sources. Website has been updated preacherbryan.com to have a forum and the podcast front and center. Just press play. Subscription are slowly being integrated. Let me know if you are not seeing Hard Verses on your favorite player. Please, don't be antisocial. Get connected: Facebook- Bryan Cook Twitter- Preacherbryan Google +- Bryan Cook Instagram wesixcooks2 Forum on preacherbryan.com

    Episode 3: The Unjust Steward, Luke 16:1-13

    Play Episode Listen Later May 19, 2016 32:57


    Be sure and download the PDF below. I promise it will help to understand the lesson. The Unjust Steward Digging Deeper into the Parable   Hard Verses Episode 3: The Unjust Steward. (Luke 16:1-13) Part 1.     The Problem. And he said also unto his disciples, There was a certain rich man, which had a steward; and the same was accused unto him that he had wasted his goods. And he called him, and said unto him, How is it that I hear this of thee? give an account of thy stewardship; for thou mayest be no longer steward. Then the steward said within himself, What shall I do? for my lord taketh away from me the stewardship: I cannot dig; to beg I am ashamed.  I am resolved what to do, that, when I am put out of the stewardship, they may receive me into their houses. So he called every one of his lord's debtors unto him, and said unto the first, How much owest thou unto my lord?  And he said, An hundred measures of oil. And he said unto him, Take thy bill, and sit down quickly, and write fifty.  Then said he to another, And how much owest thou? And he said, An hundred measures of wheat. And he said unto him, Take thy bill, and write fourscore. And the lord commended the unjust steward, because he had done wisely: for the children of this world are in their generation wiser than the children of light.  And I say unto you, Make to yourselves friends of the mammon of unrighteousness; that, when ye fail, they may receive you into everlasting habitations. He that is faithful in that which is least is faithful also in much: and he that is unjust in the least is unjust also in much.  If therefore ye have not been faithful in the unrighteous mammon, who will commit to your trust the true riches?  And if ye have not been faithful in that which is another man's, who shall give you that which is your own?  No servant can serve two masters: for either he will hate the one, and love the other; or else he will hold to the one, and despise the other. Ye cannot serve God and mammon.                                                                                                                                                                 Luke 16:1-13 KJV I have actually had people say there is nothing you can get from this parable and that Jesus would not have said this. Whoever can come up with a solution to this parable can have a dozen people shoot it down with objections. Let us look at some of these problems. Why did the lord commend the unjust steward for giving away his product? Why did the lord commend the unjust steward while he was being dishonest and going behind his back? How are the children of the world wiser than the children of light? How could Jesus say to make friends of the unrighteous mammon and that this could produce everlasting habitation (Heaven)?   The Solution. We will be looking at the different elements of the parable in historical and cultural context (See attached PDF, “The Unjust Steward: Digging Deeper into the Parable.”)     The lesson for us today. Will be discussed at a later episode.     Call to action. Download the pdf “The Unjust Steward.” This is a master’s level paper with peer reviewed sources. Website has been updated preacherbryan.com to have a forum and the podcast front and center. Just press play. Subscription are slowly being integrated. Let me know if you are not seeing Hard Verses on your favorite player. Please, don't be antisocial. Get connected: Facebook- Bryan Cook Twitter- Preacherbryan Google +- Bryan Cook Instagram wesixcooks2 Forum on preacherbryan.com

    I Came Not To Call The Righteous But Sinners; Mark 2:17

    Play Episode Listen Later May 17, 2016 19:45


    Hard Verses Episode 2: I came not to call the righteous, but the sinner... (Mark 2:17)   The Problem.   Upon first glance there are a few questions that come to mind. I thought that Jesus came to call everyone to repentance. John 3:16 and 17--That the world might be saved. God has called righteous men before. Noah (Gen. 6:9), (Job 1:1). I thought that evil communications corrupts good manners (1 Cor. 15:33). The problem from those first century Jews. The respected teacher of the law (Jesus) seemed to draw the undesirables. They asked the disciples, why is He hanging out with these people? Sinners? There was a problem with Jesus associating with Gentiles and Samaritans. Also when one calls sinners to associate and follow Him, that does not look like godly behavior.  Were they even changing their behavior?   The Solution.   Jesus came to die for all men so how it is that he is only calling the sinners and not the righteous. Many cases one could replace "righteous" with "self-righteous." Pharisees and scribes did not see themselves as in need of forgiveness. Let us remember the Pharisee and the Publican that went down to pray.  Only one recognized his sin. There has to be an acknowledgement that one is truly in need of saving. Jesus called upstanding men to follow Him and even some of the elite religious class also followed Jesus. Nicodemus was one of those men. The problem of hanging out with sinners was all about intent. He intended everyone to repent. The woman that was taken in adultery and was about to be stoned, He proclaimed that He was not going to condemn her but then told her to go and sin no more.   Jesus saw just how great a need these individuals needed a savior. Just like a doctor that sees you are having a stroke or heart attack. They see the urgency to get them well.   The lesson for us today.   Are we giving up on individuals? Sinners? Are they in less need of a savior than in the first century? Jesus is still calling the sinner today to come to repentance. Maybe we have become as self-righteous as the Pharisees and scribes that we no longer see ourselves in need of Jesus' saving power.   Call to action. Website has been updated preacherbryan.com to have a forum and the podcast front and center. Just press play. Subscription are slowly being integrated. Let me know if you are not seeing Hard Verses on your favorite player. Please, don't be antisocial. Get connected: Facebook- Bryan Cook Twitter- Preacherbryan Google +- Bryan Cook Instagram wesixcooks2 Forum on preacherbryan.com

    Hard Verses: Pilot Episode; Eat My Flesh, Drink My Blood, John 6:53

    Play Episode Listen Later May 3, 2016 15:28


    Show Notes.   Thanks to my son, Sam Cook, for the introduction.   The Problem (why is this verse considered so difficult).   Joh 6:53  Then Jesus said unto them, Verily, verily, I say unto you, Except ye eat the flesh of the Son of man, and drink his blood, ye have no life in you. Joh 6:54  Whoso eateth my flesh, and drinketh my blood, hath eternal life; and I will raise him up at the last day. Joh 6:55  For my flesh is meat indeed, and my blood is drink indeed. Joh 6:56  He that eateth my flesh, and drinketh my blood, dwelleth in me, and I in him.   Even the people during that time had a problem with this saying. Joh 6:60  Many therefore of his disciples, when they had heard this, said, This is an hard saying; who can hear it? Joh 6:61  When Jesus knew in himself that his disciples murmured at it, he said unto them, Doth this offend you?   Joh 6:66  From that time many of his disciples went back, and walked no more with him. Joh 6:67  Then said Jesus unto the twelve, Will ye also go away? Joh 6:68  Then Simon Peter answered him, Lord, to whom shall we go? thou hast the words of eternal life.    The Solution   The clue has to do with how Jesus is defining these terms. He brought up manna in the wilderness. God supplied their necessity of food with bread from heaven. Jesus states that He is the bread of heaven in which you will not die.   Joh 6:48  I am that bread of life. Joh 6:49  Your fathers did eat manna in the wilderness, and are dead. Joh 6:50  This is the bread which cometh down from heaven, that a man may eat thereof, and not die.   How does one eat of this bread?   Jesus tells us what the bread is.   Joh 6:51  I am the living bread which came down from heaven: if any man eat of this bread, he shall live for ever: and the bread that I will give is my flesh, which I will give for the life of the world.                  This is the crucifixion.   This still does not answer the question how to partake of this bread of heaven. He explains that this must be done spiritually, notice:   Joh 6:61  When Jesus knew in himself that his disciples murmured at it, he said unto them, Doth this offend you? Joh 6:62  What and if ye shall see the Son of man ascend up where he was before? Joh 6:63  It is the spirit that quickeneth; the flesh profiteth nothing: the words that I speak unto you, they are spirit, and they are life. Joh 6:64  But there are some of you that believe not. For Jesus knew from the beginning who they were that believed not, and who should betray him.   He made the point that they must believe him and that He is speaking about spiritual things.  His flesh will not be found after all the suffering and anguish.  Subscribe to the Youtube, RSS, Stitcher, iTunes, etc. Go to the website (preacherbryan.com) get show notes, graphics, other Bible studies. Got a verse that has got you stumped you can email at hardverses@preacherbryan.com or hit me up on facebook, twitter, Google plus. Or the forum.  

    Claim Hard Verses: Not Your Namby Pamby Bible Study. These Bible Verses are Hard!

    In order to claim this podcast we'll send an email to with a verification link. Simply click the link and you will be able to edit tags, request a refresh, and other features to take control of your podcast page!

    Claim Cancel